Difference between revisions of "2013 Mock AIME I Problems/Problem 14"
(I am new to latex, but briefed the solution.) |
Sugar rush (talk | contribs) |
||
Line 1: | Line 1: | ||
− | Problem | + | ==Problem== |
+ | Let <math>P(x) = x^{2013}+4x^{2012}+9x^{2011}+16x^{2010}+\cdots + 4052169x + 4056196 = \sum_{j=1}^{2014}j^2x^{2014-j}.</math> If <math>a_1, a_2, \cdots a_{2013}</math> are its roots, then compute the remainder when <math>a_1^{997}+a_2^{997}+\cdots + a_{2013}^{997}</math> is divided by 997. | ||
− | + | ==Solution== | |
− | |||
− | |||
Since 997 is prime, we have <math>a_1^{997}+a_2^{997}+\cdots + a_{2013}^{997}</math> equals to <math>a_1+a_2+\cdots + a_{2013}</math> mod 997, which by Vieta's equals -4. Thus our answer is 993 mod 997. | Since 997 is prime, we have <math>a_1^{997}+a_2^{997}+\cdots + a_{2013}^{997}</math> equals to <math>a_1+a_2+\cdots + a_{2013}</math> mod 997, which by Vieta's equals -4. Thus our answer is 993 mod 997. |
Revision as of 15:08, 15 December 2020
Problem
Let If are its roots, then compute the remainder when is divided by 997.
Solution
Since 997 is prime, we have equals to mod 997, which by Vieta's equals -4. Thus our answer is 993 mod 997.